LSAT and Law School Admissions Forum

Get expert LSAT preparation and law school admissions advice from PowerScore Test Preparation.

 Administrator
PowerScore Staff
  • PowerScore Staff
  • Posts: 8917
  • Joined: Feb 02, 2011
|
#41178
Complete Question Explanation
(The complete setup for this game can be found here: lsat/viewtopic.php?t=15645)

The correct answer choice is (C)

Answer choice (A) is incorrect because only H, L, P, G, or T can be harvested first.

Answer choice (B) is incorrect because the earliest K can be harvested is fourth.

Answer choice (D) is incorrect because the latest G can be harvested is sixth.

Answer choice (E) is incorrect because only M or J can be harvested last.
 jaxxcourt
  • Posts: 1
  • Joined: Nov 19, 2020
|
#81339
Can someone please explain to me how the latest G could be harvested is 6th? This makes no sense to me in my mind & it has been driving me crazy trying to understand this. Please explain the reasoning and order.
 Robert Carroll
PowerScore Staff
  • PowerScore Staff
  • Posts: 1787
  • Joined: Dec 06, 2013
|
#81353
jaxx,

Isolate the first and third rules - G is before K, and K is before M. So G is before at least two things. It thus can't be any later than sixth.

Note on the diagram linked in the first post here, G is shown before K and M, displaying those facts visually as well.

Robert Carroll
User avatar
 JoshuaDEL
  • Posts: 15
  • Joined: Apr 25, 2021
|
#86638
Hello,

Is the answer C because H and J can go behind M to make M go in the 6th position?
User avatar
 KelseyWoods
PowerScore Staff
  • PowerScore Staff
  • Posts: 1079
  • Joined: Jun 26, 2013
|
#86663
Hi Joshua!

Yes, M only has to have 5 fields before it: T, P, G, L, and K. J and H could be before or after M. So that means that M could go 6th.

For all of these answer choices, look at your diagram and count up how many variables have to be before and after the variable in question. J can't be first because it has to have at least 2 fields (L and H) before it. K can't be second because it has to have at least 3 fields (P, G, and L) before it. G can't be seventh because it has to have at least 2 fields (K and M) after it. T can't be 8th because it has to have at least 1 field (M) after it.

Hope this helps!

Best,
Kelsey

Get the most out of your LSAT Prep Plus subscription.

Analyze and track your performance with our Testing and Analytics Package.